Question

Find the market equilibrium point for the following demand and supply functions. Demand:        p = -4q +...

Find the market equilibrium point for the following demand and supply functions.

Demand:        p = -4q + 310

Supply:          p = 6q + 1

p = ?

2nd question:

A shoe store owner will buy 13 pairs of a certain shoe if the price is $73 per pair and 33 pairs if the price is $23. The supplier of the shoes is willing to provide 22 pairs if the price is $76 per pair but only 2 pairs if the price is $24. Assuming the supply and demand functions for the shoes are linear, find the market equilibrium point.

Part 1 of 10

Find the two ordered pairs for the demand function in the form (q, p). List your answers from smallest q to largest.
(q, p) = (? )
(q, p) = ( ? )

Homework Answers

Answer #1

Answer :

1) Demand function is p = -4q + 310

Supply function is p = 6q + 1

For the market equilibrium point set -4q + 310 = 6q + 1

10q = 309

q = 30.9

Hence , from demand equation p = -4(30.9) + 310 = 186.4

therefore , p = 186.4

2) Consider the points ( 13, 73) and ( 33, 23)

The slope is m = ( 23-73)/(33-13) = -5/2

The demand equation is

p - 73 = -(5/2)(q - 13)

p = -(5q/2) +(211/2)

Consider the points (22 , 76) and ( 2, 24)

The slope is m= ( 24-76)/(2-22) = 13/5

The supply equation is  

p - 76 =(13/5)(q-22)

p = (13q/5) + (94/5)

For the market equilibrium point set

-(5q/2) +(211/2) = (13q/5) + (94/5)

Multiply by 10 on both sides , we get

-25q + 1055 = 26q + 188

51q = 867

q = 17 and so from demand equation p =63

The market equilibrium point is ( q , p ) = ( 17 , 63)

Know the answer?
Your Answer:

Post as a guest

Your Name:

What's your source?

Earn Coins

Coins can be redeemed for fabulous gifts.

Not the answer you're looking for?
Ask your own homework help question
Similar Questions
The demand and supply equations for the Wheat market are: Demand: P = 200-4q Supply: P...
The demand and supply equations for the Wheat market are: Demand: P = 200-4q Supply: P = - 50 + Q Where P = price per bushel, and Q = quantity 1. Calculate the equilibrium price and quantity. (1.5 Marks) 2. Suppose the government guaranteed producers a price floor of AED 90 per bushel. Estimate the effect on the quantity supplied and demanded. (1.5 Marks) 3. Would the price floor affect the Market outcome? (Calculate the surplus or shortage )....
Find the equilibrium quantity and equilibrium price for the commodity whose supply and demand functions are...
Find the equilibrium quantity and equilibrium price for the commodity whose supply and demand functions are given. ​Supply:p=120q  Demand: p= - q2 +16,000 The equilibrium quantity is q = ------------ at price p=​ $ -------------.
The market for a product has inverse demand and supply functions given by p = 290...
The market for a product has inverse demand and supply functions given by p = 290 - 2Qd and p = 10 + 1.5Qs In what form are these functions in? (2pts) Find the market equilibrium quantity Q* and price P*. (5pts) Draw out a simple graph with these curves. Label the p-intercept for each and indicate the equilibrium points. (5pts) Find the consumer and producer surpluses, along with the total surplus.(10pts) (i) Would this market be considered efficient? (2pts)
In a given market, supply is given by the equation p = q^2 + 20, and...
In a given market, supply is given by the equation p = q^2 + 20, and the demand is given by the equation p = 140 − 3q^2 − 6q. A tax of $42 is placed on the production of the market’s product. Find the after-tax equilibrium point for this market, that is, both the quantity and the unit price in equilibrium.
Estimate the equilibrium price and quantity of the market whose demand and supply functions are pd...
Estimate the equilibrium price and quantity of the market whose demand and supply functions are pd = - (q + 4)^2 + 100 and ps = (q + 2)^2 respectively
Suppose the demand and supply for a product is given by the following equations: p=d(q)=−0.8q+150 (Demand)...
Suppose the demand and supply for a product is given by the following equations: p=d(q)=−0.8q+150 (Demand) p=s(q)=5.2q (Supply) For both functions, q is the quantity and p is the price. Find the equilibrium point. (Equilibrium price and equilibrium quantity) (1.5 Marks) Compute the consumer surplus. (1.5 Marks) Compute the producer surplus. (1.5 Marks)
1) The demand and supply functions for a certain product are given by p = 150...
1) The demand and supply functions for a certain product are given by p = 150 − 0.5 q and p = 0.002 q^ 2 + 1.5 where p is in dollars and q is the number of items. a) Which is the demand function? Why do you know? b) Find the equilibrium price and quantity. c) Find the total gains from trade at the equilibrium price.
The supply and demand for lobster at the K. Lopez Fish Store are related by the...
The supply and demand for lobster at the K. Lopez Fish Store are related by the equations: Supply: p=S(q)=6q+3 and Demand: p=D(q)=-2q+19 where q represents the quantity of lobster in pounds per day and p represents the price in dollars per pound. (a) Identify your random variable(s) (b) Graph both the supply and demand functions on the same axes. (c) Estimate the equilibrium quantity and the equilibrium price from your graph. (d) Find the equilibrium quantity (solve) (e) Find the...
The general demand and supply functions given below describe the market for coffee. Demand and Supply...
The general demand and supply functions given below describe the market for coffee. Demand and Supply are both functions of coffee's own price, P. Supply is also a function of the price of an input, coffee beans, Pb. Qd=D(P) Qs=S(P,Pb) a.) Use comparative statics to predict how the equilibrium price P*, changes when Pb increases. Start with the equilibrium condition, incorporating the fact that the equilibrium price, P*, is a function of Pb so that D(P*(Pb))=S(P*(Pb),Pb). Put a sign on...
Suppose the global Soybean market is competitive and currently has the following supply and demand functions:...
Suppose the global Soybean market is competitive and currently has the following supply and demand functions: QD = 700 – 0.5PS and QS = PS – 500. The market expects to see a 25% increase in the market price within a year due to change in demand. What will be the new equilibrium price and equilibrium quantity of the market keeping all other things constant? New P*= New Q*=